Download as pdf or txt
Download as pdf or txt
You are on page 1of 8

EIGHTEENTH IRISH MATHEMATICAL OLYMPIAD

Saturday, 7 May 2005

10 a.m.–1 p.m.

First Paper

1. Prove that 20052005 is a sum of two perfect squares, but not the sum of two
perfect cubes.

2. Let ABC be a triangle and let D, E and F , respectively, be points on the sides
BC, CA and AB, respectively—none of which coincides with a vertex of the
triangle—such that AD, BE and CF meet at a point G. Suppose the triangles
AGE, CGD and BGF have equal area. Prove that G is the centroid of ABC.

3. Prove that the sum of the lengths of the medians of a triangle is at least three
quarters of the sum of the lengths of the sides.

4. Determine the number of different arrangements a1 , a2 , . . . , a10 of the integers


1, 2, . . . , 10 such that
ai > a2i for 1 ≤ i ≤ 5,
and
ai > a2i+1 for 1 ≤ i ≤ 4.

5. Suppose a, b and c are non-negative real numbers. Prove that


1 √
3
[(a−b)2 +(b−c)2 +(c−a)2 ] ≤ a2 +b2 +c2 −3 a2 b2 c2 ≤ (a−b)2 +(b−c)2 +(c−a)2 .
3
OUTLINE SOLUTIONS

1. Prove that 20052005 is a sum of two perfect squares, but not the sum of two
perfect cubes. [GL]

Solution. 2005 = 5 × 401. 5 = 22 + 12 and 401 = 202 + 12 . Using the fact


that the modulus of a product of two complex numbers is the product of their
moduli, or the formula

(a2 + b2 )(c2 + d2 ) = (ac − bd)2 + (bc + ad)2 ,


we find 2005 = 412 + 182 = 392 + 222 .
Then

20052005 = 2005 × 20052004 = (412 + 182 )(20051002 )2 = a2 + b2

where a = 41 × 20051002 and b = 18 × 20051002 .


Alternatively, we apply the formula repeatedly to express 20052005 as a sum of
two squares.

(Since the set of sums of squares {a2 + b2 : a, b ∈ N} is closed under multiplica-


tion (?), 2005n belongs to this set for any natural number n. Also, as Fiachra
Knox observed, it is a standard fact (due to Euler (?)) that an integer belongs
to this set iff if it is of the form xy 2 , where x, y ∈ N and x has no prime factor
of the form 4k + 3. Thus every prime congruent to 1 modulo 4 belongs to the
set, a fact that was proved/conjectured (?) by Fermat.)
Cubes are 0, 1 or −1 modulo 7. Thus a sum of two cubes is 0, 1, 2, −1, −2
modulo 7. 2005 = 3mod7. Hence 20052005 = 32005 mod7 = 36.334 × 3 = 3mod7.
Hence 20052005 cannot be a sum of two cubes. A mod 13 argument will also
work.

2. Let ABC be a triangle and let D, E and F , respectively, be points on the sides
BC, CA and AB, respectively—none of which coincides with a vertex of the
triangle—such that AD, BE and CF meet at a point G. Suppose the triangles
AGE, CGD and BGF have equal area. Prove that G is the centroid of ABC.
[JL]

Solution. Let 4P QR stand for the area of a triangle P QR. In this notation,
let

a = 4AGE, x = 4EGC, b = 4CGD, y = 4DGB, c = 4BGF, z = 4F GA.

Then
4AGB AG 4AGC
= = ,
4ADB AD 4ADC
i.e.,
z+c AG a+x
= = ,
z+c+y AD a+x+b
or, inverting this,
y b
= .
z+c a+x
Hence

ay + xy = bz + bc. (1)
A

F E
G

B D C

Similarly,

cx + zx = ay + ab, bz + yz = cx + ca. (2)

Adding these equations we deduce that

ab + bc + ca = xy + yz + zx.

Ceva’s Theorem implies


abc
= 1, abc = xyz (3)
xyz
These facts are independent of the hypotheses. If we now bring these into play,
we can assume that a = b = c = 1 in which case xyz = 1 and xy + yz + zx = 3.
Thus the geometric mean and the arithmetic mean of xy, yz, zx are equal.
Hence x = y = z = 1 and the result follows.
Alternatively, by (1) and (2), z +1 = y(x+1), y +1 = x(z +1) and eliminating
x and y, we obtain z 4 + 3z 3 − z 2 − 4z + 1 = 0. Factorising the quartic we
obtain (z − 1)(z 3 + 4z 2 + 3z − 1) = 0. Hence z = 1 or z 3 + 4z 2 + 3z − 1 = 0.
The cubic has a single real root between 0 and 1. x and y satisfy the same
equation. But x, y, z > 0 and xyz = 1. This is only possible if x = y = z = 1.
D, E, F are then the mid points of BC, CA and CB. Hence G is the centroid.

3. Prove that the sum of the lengths of the medians of a triangle is at least three
quarters of the sum of the lengths of the sides. [TL]

Solution. Let ma , mb , mc denote the lengths of the medians of a triangle ABC


with centroid G. Then ma = 3|AG|/2, mb = 3|BG|/2, mc = 3|CG|/2, and
their sum is given by
3
ma + mb + mc = [|AG| + |BG| + |CG|]
2
3
= ((|AG| + |BG|) + (|BG| + |CG|) + (|CG| + |AG|))
4
3
> (|AB| + |BC| + |CA|),
4
by three applications of the triangle inequality.
Remark.

|AG| + |BG| + |CG| ≥ min [|AP | + |BP | + |CP |] = |AF | + |BF | + |CF |,
P ∈ABC

where F is the Fermat-Torricelli point. Equality holds here iff the triangle is
equilateral.
Alternatively, denoting the vertices of the triangle by vectors a, b, c, then, by
definition,
b+c c+a a+b
ma = | − a|, mb = | − b|, mc = | − c|.
2 2 2
Hence
3 a+b+c
ma + mb + mc = [|m − a| + |m − b| + |m − c|] (m = )
2 3
3
= [(|m − a| + |m − b|) + (|m − b| + |m − c|)
4
+ (|m − c| + |m − a|)]
3
> [|a − b| + |b − c| + |c − a|].
4
Another approach would be to determine expressions for the medians in terms
of the side lengths. Two applications of the Cosine Rule tell us that

4m2a = 2(b2 + c2 ) − a2 = b2 + c2 + 2bc cos A = |b + ceiA |2 ,


with similar formulae for mb , mc . Thus
X X X Xp
4 m2a = 3 a2 , 2 ma = 2(b2 + c2 ) − a2 .

While it’s a short step to see from the first identity that
X 1
ma > (a + b + c),
2
this appears to be the best that one can deduce from it. And working with
the second seems a hopeless task!

Note too that the constant 3/4 is sharp. To see this, let ABC be a triangle
with b = c = 1, a = 2, 0 <  < 1 so that
√ √
2(ma + mb + mc ) 1 − 2 + 1 + 82
=
a+b+c 1 +s

r
1− 1 + 82
= +
1+ (1 + )2
r
1− 3
< + ,
1+ 2
since
9 1 + 82 (1 − )(5 + 23)
− 2
= > 0.
4 (1 + ) (1 + )2

It follows from this that there is no absolute constant ν > 3/4 such that

ma + mb + mc ≥ ν(a + b + c),

for all triangles.


There are also direct and successful approaches using the cosine and sine rules.
Here’s something else that you might like to try your hands at:

Problem 1 Prove that

ma + mb + mc < a + b + c.

Prove also that this is sharp.

Problem 2 Prove that ma , mb , mc are the side lengths of a triangle—called the


median dual triangle of ABC—the lengths of whose medians are 3a/4, 3b/4, 3c/4.
Problem 3 Prove that

ma + mb + mc ≥ a sin A + b sin B + c sin C,

with equality iff the triangle is equilateral.

Problem 4 Prove that the sum of the lengths of the medians of a triangle is at
least three times the sum of the lengths of the perpendiculars from the centroid
to the sides. Prove also that there is equality iff the triangle is equilateral.

4. Determine the number of different arrangements a1 , a2 , . . . , a10 of the integers


1, 2, . . . , 10 such that
ai > a2i for 1 ≤ i ≤ 5,
and
ai > a2i+1 for 1 ≤ i ≤ 4. [DH]

Solution. Suppose {a1 , a2 , a3 , a4 , a5 , a6 , a7 , a8 , a9 , a10 } is such an arrangement.


Then the inequalities that hold can be represented by the following tree-
diagram where ai → aj means ai > aj : Clearly, the tree has two main branches,

a_1

a_3
a_2
a_7
a_4
a_5
a_8 a_6
a_9 a_10

one bifurcating at a2 and the other at a3 . Since a1 is the largest, a1 = 10.


So, to count the number of different arrangements subdivide the numbers
1, 2, . . . , 9 into two sets {a2 , a4 , a5 , a8 , a9 , a10 } and {a3 , a6 , a7 }. We can carry
out this subdivision in 93 = 84 ways. For each subdivision, a2 and a3 are
uniquely determined as the largest elements in their subsets. For the choice a6
and a7 we have two possibilities, and for the subsets {a4 , a8 , a9 } and {a5 , a10 }
we have 52 = 10 ways to do it. For each of these choices, a4 , a5 , a10 are already
determined. There are again two possibilities for a8 and a9 . Thus the number
of possibilities to arrange the full set as required is
   
9 5
×2× × 2 = 84 × 2 × 10 × 2 = 3360
3 2
ways.

5. Suppose a, b and c are non-negative real numbers. Prove that


1 √
3
[(a−b)2 +(b−c)2 +(c−a)2 ] ≤ a2 +b2 +c2 −3 a2 b2 c2 ≤ (a−b)2 +(b−c)2 +(c−a)2 . [FH]
3

Solution. Noting that

(a − b)2 + (b − c)2 + (c − a)2 = 3(a2 + b2 + c2 ) − (a + b + c)2 ,

the left-hand inequality follows from the fact that


√  √ 2  2
3
2 2 2 3 a+b+c (a + b + c)2
3 a b c =3 abc ≤ 3 = .
3 3
To establish the right-hand inequality we must show that

3
2(ab + bc + ca) ≤ a2 + b2 + c2 + 3 a2 b2 c2 .

Since the expressions are symmetric we can assume without loss of generality
that a is the minimum of {a, b, c}. With this assumption,

3

3
a2 + b2 + c2 + 3 a2 b2 c2 = a2 + 3 a2 b2 c2 + b2 + c2
p
≥ 4 4 a2 (a2 b2 c2 ) + b2 + c2

= 4a bc + (b − c)2 + 2bc

= 2a(2 bc − (b + c)) + (b − c)2 + 2(ab + bc + ca)
√ √
= −2( c − b)2 a + (c − b)2 + 2(ab + bc + ca)
√ √ √ √ 
= ( c − b)2 ( c + b)2 − 2a + 2(ab + bc + ca)
≥ 2(ab + bc + ca),

since √ √
( c + b)2 − 2a ≥ 4a − 2a = 2a ≥ 0,
with equality iff a = 0.
As Fiachra Knox observed—and Stephen Ryan wondered about—Schur’s in-
equality to the effect that if r ≥ 0 and x, y, z are positive numbers, then

xr (x − y)(x − z) + y r (y − z)(y − x) + z r (z − x)(z − y) > 0,


unless x = y = z can be used to establish the right-hand side. [Hint: Take r =
1, expand and rearrange this inequality and set x = a2/3 , y = b2/3 , z = c2/3 .]
Also, as Richard Ellard essentially noted, the left-hand side can be strength-
ened:

Problem 5 Prove that


1 √
3
[(a − b)2 + (b − c)2 + (c − a)2 ] ≤ a2 + b2 + c2 − 3 a2 b2 c2 .
2

Problem 6 Prove Schur’s inequality.

You might also like